PT8.S1.Q9- Milk Bottlers

miriaml7miriaml7 Live Member
edited July 2020 in Logical Reasoning 1014 karma

I am really struggling with this question and I honestly think it's because I may have misunderstood the stimulus. My understanding is that milk, the final product that we see at the store, requires the use of a bottle and raw milk. The bottler is the person who purchases raw milk from farmers and uses it to create the jug of milk that we see at the grocery store. The stimulus proceeds to tell us that complaints about the bottlers pulling up the prices tend to happen when the milk bottler has had to purchase raw milk from farmers, at a higher price than usual. The author of the stimulus uses this to show that it's not the milk bottler's fault that the price of milk at the grocery store has gone up, but rather the blame should be on the farmer. If the farmer hadn't pulled up the price of raw milk then the bottler would not have had to pull up the price of their final product (i.e. the milk jug at the store).
The word "markup" was something I interpreted to mean revenue, not sure if that's actually what was meant by that. From that I drew out:

Scenario 1: Price of raw milk goes up

Price of milk at grocery store:
Goes up

Markup:
small

Scenario 2: Price of raw milk falls

Price of milk at grocery store:
Goes down

Markup:

greatest

I ended up choosing answer choice D, but it was not the accredited answer. I would greatly appreciate if someone could read through my interpretation of the stimulus and help me figure out where I went wrong.

Thanks in advance!

Comments

  • OneFortyDotSixOneFortyDotSix Alum Member
    edited July 2020 634 karma

    Mark-up isn't precisely revenue, it's really margin. If you're buying widgets for $5 and selling them for $7, your markup is $2. Per widget your cost is $5, and your revenue is $7

    Based on that choice D appears to be correct - the key word here is 'proportionately'. The last few lines of the stim state that as the price of rawmilk (bottlers' cost) is rising, their markup is actually smallest proportionate to the retail price. The fact that mark-ups are greater when the rawmilk price is falling means that bottlers' aren't reducing their sales price in proportion to what they're paying for rawmilk when rawmilk prices are falling.

    Answer D states this precisely.

    C is a very attractive trap answer. You can maybe make a case for it if this was an MSS question but for an MBT the support needs to be ironclad as it is for choice D.

  • Chris NguyenChris Nguyen Alum Member Administrator Sage 7Sage Tutor
    edited July 2020 4577 karma

    Hey Miriam!

    So the answer to this question is actually what you chose, answer choice D :smile:

    I think your reasoning is great, the only thing is that markup is not the same thing a revenue. The markup is the difference between the actual cost of what is being supplied and what the company decides to sell it at. Revenue is all the money you get from selling your product (not taking into account costs. Profit is when you take into account your costs).

    Edit: Just saw that @OneFortyDotSix posted while I was writing up my post so I didn't see it! I think his explanation is spot on!

    Another thing to add about C - the stimulus only gives us information about when the price is rising or falling, but never when it stays stagnant at a low price or a high price. It could be the case when it stays stagnant that they would eventually take a higher markup when the price stays high. We just don't know.

  • miriaml7miriaml7 Live Member
    1014 karma

    Thank you both!! @OneFortyDotSix @Christopherr

  • NamritaktNamritakt Member
    175 karma

    Regarding AC D, if the raw milk price and bottled milk price decrease proportionately, then there is no net change in the markup.
    For example:
    $10 retail price - $5 raw milk price = 50% mark up
    Following a proportional 10% decrease:
    $9 retail price - $4.50 = 50% mark up.

    Let’s compare this to when raw milk prices rise: $10 retail price -$9 raw milk price = 10% mark up.
    So considering that a proportional decrease in bottled milk prices could result in a 50% (unchanged) markup compared to a 10% markup when prices rise, is it not possible that even when raw milk prices fall and bottled milk prices fall proportionately, there can still be the greatest markups (making AC D not MBT) I know there obviously something wrong with my reasoning.but I would appreciate some help! @Christopherr @OneFortyDotSix

  • Chris NguyenChris Nguyen Alum Member Administrator Sage 7Sage Tutor
    edited July 2020 4577 karma

    Hey @taggarnk!

    I think your reasoning is great! But D is saying that milk bottlers generally do not respond to raw milk price decreases by proportionately decreasing the price.

    I think you're saying (D) says milk bottlers generally do decrease the price proportionately :smile:

    It's super easy to gloss over a "not" and it's something that we have to be very wary of when doing lsat questions (I've definitely done it too many times to count)

    Let me know if i'm wrong in thinking this, and if I can help in any other way!

  • NamritaktNamritakt Member
    edited July 2020 175 karma

    @Christopherr thank you so much for the response! I actually did read the “not”. I was trying to demonstrate that bottlers could respond to lower raw milk prices by lowering bottled milk prices proportionately (thereby eliminating this AC as a potential MBT), without conflicting with the condition that “when the raw-milk price is falling, however, the markups are greatest”.To do this, I showed that although lowering prices proportionately results in no net change in the mark up (still 50%, as illustrated above) this resulting markup value could still be the greatest relative to when milk prices rise (10% markup in example above), which is what we need according to the following:
    “When the raw-milk price is falling, however, the markups are greatest”

    I know that my reasoning is flawed seeing as AC D a MBT, but I cannot seem to pinpoint why 😔. Thanks for your help!!!

  • Chris NguyenChris Nguyen Alum Member Administrator Sage 7Sage Tutor
    edited July 2020 4577 karma

    @taggarnk I apologize about misinterpreting!

    I PMed you! Hopefully we can get to the bottom of this.

Sign In or Register to comment.